0 Daumen
711 Aufrufe

Es liegen zwei Ereignisse A und B vor, mit P(A)=0,4 und P(¯A ∩B)= 0,24 und ich soll prüfen ob das stochastisch unabhängig ist. 

Ich muss also testen, ob P(¯A∩B)=P(¯A) * P(B) ist. 

Dass P(¯A)=0,6 ist, ist klar. Aber wie berechne ich P(B)? Fülle ich das Baumdiagramm aus, bekomme ich ja nur bedingte Wahrscheinlichkeiten, aber nicht P(B)? 

Weiß jemand Rat?

Avatar von

(1.) Der Ausdruck "linear unabhängig" hat hier nichts zu suchen. Es geht um stochastische Unabhängigkeit.

(2.) Ich denke, dass die vorliegenden Angaben nicht genügen, um die Unabhängigkeit nachzuweisen oder zu widerlegen.

(1) Ja, tut mir leid, da habe ich mich verschrieben. Ich meine natürlich stochastisch unabhängig 

(2) Das war auch schon meine Vermutung...

Vom Duplikat:

Titel: Übersetzen Sie folgendes Baumdiagramm in eine Vierfeldertafel

Stichworte: vierfeldertafel,baumdiagramm,stochastik

Aufgabe:

5 a) Übersetzen Sie das nachstehende Baumdiagramm in eine Vierfeldertafel.

b) Erstellen Sie ein Baumdiagramm, in dem C und D auf der ersten Stufe stehen.

c) Bestimmen Sie die bedingten Wahrscheinlichkeiten \( \mathrm{P}_{\mathrm{A}}(\mathrm{C}) \) und \( \mathrm{P}_{\mathrm{C}}(\mathrm{A}) \) und formulieren Sie eine Aussage im Kontext von Haarfarben.

d) Untersuchen Sie A und D auf Unabhängigkeit.

1 Antwort

0 Daumen

" Fülle ich das Baumdiagramm aus, bekomme ich ja nur bedingte Wahrscheinlichkeiten, aber nicht P(B)?  "

Schau, wo überall B rauskommt und addiere alles.

Alternativ: Vierfeldertafel:  Vergrössere die Tabelle und berechne die Summen von Zeilen und Spalten.

" ich soll prüfen ob das stochastisch unabhängig ist.  " Meinst du mit "das" A und B oder etwas anderes? 

Möchtest du auf

Skärmavbild 2020-06-22 kl. 09.33.32.png

Text erkannt:

Qemerkung I \( \Leftrightarrow \) Der_Mathecoach I "Frage mit Sicherheit unvollständig"

reagieren?

P(A)=0,4 und P(¯A ∩B)= 0,24 und ich soll prüfen ob das stochastisch unabhängig ist. 

Ich muss also testen, ob P(¯A∩B)=P(¯A) * P(B) ist. 

Dass P(¯A)=0,6 ist, ist klar.


Bei Unabhängigkeit könnte man ja einsetzen:

P(¯A∩B)=P(¯A) * P(B)

Einsetzen:

0.24 = 0.6 * P(B)

0.24 / 0.6 = P(B)

Also vielleicht P(B) = 0.4.

Nun die Fragestellung noch genauer anschauen.

Avatar von 162 k 🚀

Ein anderes Problem?

Stell deine Frage

Willkommen bei der Mathelounge! Stell deine Frage einfach und kostenlos

x
Made by a lovely community